Use app×
Join Bloom Tuition
One on One Online Tuition
JEE MAIN 2025 Foundation Course
NEET 2025 Foundation Course
CLASS 12 FOUNDATION COURSE
CLASS 10 FOUNDATION COURSE
CLASS 9 FOUNDATION COURSE
CLASS 8 FOUNDATION COURSE
0 votes
62 views
in Physics by (72.7k points)
closed by

Which of the following terms in the equation below is displacement current?

\(\oint B \cdot dl = {\mu _0}\left( {I + {\epsilon_0}\frac{{d{E}}}{{dt}}} \right)\)
1. \(\oint B \cdot dl\)
2. \(\epsilon{_0}\frac{{d{\phi _E}}}{{dt}}\)
3. \(I + {\epsilon_0}\frac{{d{\phi _E}}}{{dt}}\)
4. μ0 I

1 Answer

0 votes
by (121k points)
selected by
 
Best answer
Correct Answer - Option 2 : \(\epsilon{_0}\frac{{d{\phi _E}}}{{dt}}\)

Amperes law:

The line integral of the magnetic field around any closed curve is equal to μ0 times the net current I threading through the area enclosed by the curve.

\(\oint \vec B \cdot \overrightarrow {dl} = {\mu _0}I\)

Drawback: Ampere’s law in this form is not valid if the electric field at the surface varies with time.

Displacement current (ID): It is the current that comes into existence in addition to the conduction current, whenever the electric field and hence the electric flux changes with time.

Modified Ampere's Law:

  • To modify Ampere’s law, Maxwell followed a symmetry consideration, i.e. by Faraday’s law, a changing magnetic field induces an electric field, hence a changing electric field must induce a magnetic field.
  • As currents are the usual sources of the magnetic field, a changing electric field must be associated with the current. Maxwell called that current as displacement current.
  • To maintain the dimensional consistency, the displacement current is added in ampere’s law.

 

\(\oint \vec B \cdot \overrightarrow {dl} = {\mu _0}I + {\mu _0}{\epsilon_0}\left( {\frac{{d{{\rm{E}}}}}{{dt}}} \right)\)

\(\oint \vec B \cdot \overrightarrow {dl} = {\mu _0}[I + {\epsilon_0}\left( {\frac{{d{{\rm{E}}}}}{{dt}}} \right)]\)

Where:

\({\epsilon_0}\left( {\frac{{d{{\rm{E}}}}}{{dt}}} \right)\) is the displacement current.

Applying Stokes theorem, the above form can be written as:

\(\nabla \times B=\mu _0J+\mu _0\epsilon _0\frac{\partial E}{\partial t}\)

Welcome to Sarthaks eConnect: A unique platform where students can interact with teachers/experts/students to get solutions to their queries. Students (upto class 10+2) preparing for All Government Exams, CBSE Board Exam, ICSE Board Exam, State Board Exam, JEE (Mains+Advance) and NEET can ask questions from any subject and get quick answers by subject teachers/ experts/mentors/students.

Categories

...